Hostname: page-component-78c5997874-ndw9j Total loading time: 0 Render date: 2024-10-28T19:30:11.077Z Has data issue: false hasContentIssue false

A SHARP UPPER BOUND FOR THE SUM OF RECIPROCALS OF LEAST COMMON MULTIPLES II

Published online by Cambridge University Press:  09 June 2022

SIAO HONG*
Affiliation:
Department of Mathematics and Statistics, Brock University, St. Catharines, ON L2S 3A1, Canada
MEILING HUANG
Affiliation:
Department of Mathematics and Statistics, Brock University, St. Catharines, ON L2S 3A1, Canada e-mail: [email protected]
YUANLIN LI
Affiliation:
Department of Mathematics and Statistics, Brock University, St. Catharines, ON L2S 3A1, Canada e-mail: [email protected]
*
Rights & Permissions [Opens in a new window]

Abstract

Let n and k be positive integers with $n\ge k+1$ and let $\{a_i\}_{i=1}^n$ be a strictly increasing sequence of positive integers. Let $S_{n, k}:=\sum _{i=1}^{n-k} {1}/{\mathrm {lcm}(a_{i},a_{i+k})}$ . In 1978, Borwein [‘A sum of reciprocals of least common multiples’, Canad. Math. Bull. 20 (1978), 117–118] confirmed a conjecture of Erdős by showing that $S_{n,1}\le 1-{1}/{2^{n-1}}$ . Hong [‘A sharp upper bound for the sum of reciprocals of least common multiples’, Acta Math. Hungar. 160 (2020), 360–375] improved Borwein’s upper bound to $S_{n,1}\le {a_{1}}^{-1}(1-{1}/{2^{n-1}})$ and derived optimal upper bounds for $S_{n,2}$ and $S_{n,3}$ . In this paper, we present a sharp upper bound for $S_{n,4}$ and characterise the sequences $\{a_i\}_{i=1}^n$ for which the upper bound is attained.

Type
Research Article
Copyright
© The Author(s), 2022. Published by Cambridge University Press on behalf of Australian Mathematical Publishing Association Inc.

1 Introduction

Chebyshev [Reference Chebyshev4] investigated the least common multiple of consecutive positive integers when he made the first important attempt to prove the prime number theorem stating that $\log \mathrm {lcm}(1, 2, \ldots , n)\sim n$ as n goes to infinity (see, for example, [Reference Ireland and Rosen13]). Hanson [Reference Hanson8] and Nair [Reference Nair14] gave upper and lower bounds for $\mathrm {lcm}(1,2,\ldots ,n)$ and Nair’s lower bound was extended in [Reference Farhi6, Reference Hong, Luo, Qian and Wang11]. Goutziers [Reference Goutziers7] studied the asymptotic behaviour of the least common multiple of a set of integers not exceeding N. Bateman et al. [Reference Bateman, Kalb and Stenger1] obtained an asymptotic estimate for the least common multiple of arithmetic progressions that is generalised in [Reference Hong, Qian and Tan12] to products of linear polynomials. In another direction, Behrend [Reference Behrend2] strengthened an inequality of Heilbronn [Reference Heilbronn9] and Rohrbach [Reference Rohrbach15]. Erdős and Selfridge [Reference Erdős and Selfridge5] proved a remarkable old conjecture that predicts that the product of any two or more consecutive positive integers is never a perfect power.

Erdős observed another interesting phenomena related to least common multiples. Let n and k be positive integers with $n\ge k+1$ and let $\{a_i\}_{i=1}^n$ be a strictly increasing sequence of positive integers. Let

$$ \begin{align*} S_{n, k}:=\sum_{i=1}^{n-k}\frac{1}{\mathrm{lcm}(a_{i},a_{i+k})}. \end{align*} $$

In 1978, Borwein [Reference Borwein3] confirmed a conjecture of Erdős by showing that $S_{n, 1}\le 1-{1}/{2^{n-1}}$ with equality if and only if $a_{i}=2^{i-1}$ for $1\le i \le n$ . Recently, Hong [Reference Hong10] improved this upper bound and used the new result to get sharp upper bounds for $S_{n, 2}$ and $S_{n, 3}$ . He also characterised the sequences $\{a_i\}_{i=1}^\infty $ for which these upper bounds are attained. In this paper, we concentrate on $S_{n,4}$ . We will present an optimal upper bound for $S_{n, 4}$ and characterise the sequences $\{a_i\}_{i=1}^n$ for which this upper bound is attained.

As usual, for any real number x, we denote by $\lfloor x\rfloor $ and $\lceil x\rceil $ respectively the largest integer no more than x and the smallest integer no less than x. For brevity, we write $S_n :=S_{n, 4}$ .

The main result of this paper can be stated as follows.

Theorem 1.1. Let n be an integer with $n\ge 5$ and let $\{a_i\}_{i=1}^n$ be a strictly increasing sequence of positive integers. Then:

  1. (i) $S_{5}\le 1/5$ with equality if and only if $a_i=i$ for all $i\in \{1, 2, 3, 4, 5\}$ ;

  2. (ii) $S_{6}\le {11}/{30}$ with equality if and only if $a_i=i$ for all $i\in \{1, 2, 3, 4, 5, 6\}$ ;

  3. (iii) $S_{7}\le {43}/{90}$ with equality if and only if $a_i=i$ for all $i\in \{1, 2, 3, 4, 5, 6\}$ and $a_7=9$ ;

  4. (vi) $S_{8}\le {101}/{180}$ with equality if and only if $a_i=i$ for all $i\in \{1, 2, 3, 4, 5, 6\}$ , $a_7=9$ and $a_8=12$ ;

  5. (v) if $n\ge 9$ , then

    (1.1) $$ \begin{align} S_{n}\le \frac{493}{420}-\frac{533}{105}\cdot\frac{1} {2^{\lfloor{n}/{4}\rfloor+1}} +\frac{\epsilon_{n}}{2^{\lfloor{n}/{4}\rfloor}}, \end{align} $$
    where
    $$ \begin{align*} \epsilon_n:= {\left\{\begin{array}{rl} 0 & \text{if} \ n\equiv 0\pmod 4,\\[5pt] \frac{2}{5} & \text{if} \ n\equiv 1\pmod 4,\\[5pt] \frac{11}{15} & \text{if} \ n\equiv 2\pmod 4,\\[5pt] \frac{107}{105} & \text{if} \ n\equiv 3\pmod 4, \end{array} \right.} \end{align*} $$
    and equality in (1.1) occurs if and only if $a_i=i$ for all $i\in \{1, 2, 3, 4\}$ and $a_{4i+1}=5\times 2^{i-1} \ (1\le i\le \lfloor {(n-1)}/{4}\rfloor ), a_{4i+2}= 3\times 2^{i} \ (1\le i\le \lfloor {(n-2)}/{4}\rfloor )$ , $a_{4i+3}=7\times 2^{i-1} \ (1\le i\le \lfloor {(n-3)}/{4}\rfloor )$ and $a_{4i+4}=2^{i+2} \ (1\le i\le \lfloor {n}/{4}\rfloor -1)$ .

The rest of the paper is organised as follows. In Section 2, we prove several preliminary lemmas. In Section 3, we provide a proof for our main result.

2 Auxiliary lemmas

In this section, we supply several auxiliary lemmas that are needed in the proof of Theorem 1.1. The first is Hong’s upper bound [Reference Hong10, Theorem 1.2] which improves Borwein’s upper bound [Reference Borwein3].

Lemma 2.1 [Reference Hong10, Theorem 1.2]

Let n be an integer with $n\ge 2$ and let $\{a_i\}_{i=1}^n$ be a strictly increasing sequence of positive integers. Then

(2.1) $$ \begin{align} \sum_{i=1}^{n-1} \frac{1}{\mathrm{lcm}(a_{i},a_{i+1})} \le \frac{1}{a_{1}}\bigg(1-\frac{1}{2^{n-1}}\bigg) \end{align} $$

with equality in (2.1) if and only if $a_{i}=2^{i-1}a_{1}$ for all integers i with $1 \le i \le n$ .

Lemma 2.2. Let m be an integer with $m\ge 3$ . Then

$$ \begin{align*} \frac{1}{7}+\frac{1}{9}+\frac{1}{9}\bigg(1-\frac{1}{2^{m-2}}\bigg) <\frac{1}{5}+\frac{1}{21}+\frac{1}{5}\bigg(1-\frac{1}{2^{m-2}}\bigg) \end{align*} $$

and

$$ \begin{align*} \frac{1}{9}+\frac{1}{12}+\bigg(\frac{1}{9}+\frac{1}{10}\bigg)\bigg(1-\frac{1}{2^{m-2}}\bigg) <\frac{1}{8}+\frac{1}{21}+\bigg(\frac{1}{7}+\frac{1}{8}\bigg)\bigg(1-\frac{1}{2^{m-2}}\bigg). \end{align*} $$

Proof. Since $m\ge 3$ , a direct computation gives the desired inequalities.

Lemma 2.3. Let $S_n$ be given as above. Then:

  1. (i) $S_5\le 1/5$ with equality if and only if $a_i=i$ for all $i\in \{1, 2, 3, 4, 5\}$ ;

  2. (ii) $S_6\le {11}/{30}$ with equality if and only if $a_i=i$ for all $i\in \{1, 2, 3, 4, 5, 6\}$ ;

  3. (iii) $S_7\le {43}/{90}$ with equality if and only if $a_i=i$ for all $i\in \{1, 2, 3, 4, 5, 6\}$ and $a_7=9$ .

Proof. We first deal with $S_5$ . Since $\mathrm {lcm}(a_1, a_{4})\ge a_{5}\ge 5$ ,

(2.2) $$ \begin{align} S_{5}=\frac{1}{\mathrm{lcm}(a_1, a_{5})}\le \frac{1}{5}. \end{align} $$

The equality in (2.2) holds if and only if $\mathrm {lcm}(a_1, a_{5})=5$ , which is true if and only if $a_1=1$ and $a_5=5$ . However, $a_1<a_2<a_3<a_4<a_5$ . So the equality in (2.2) holds if and only if $a_i=i$ for all $i\in \{1,2,3,4,5\}$ .

Now consider $S_6$ . Since $a_2\ge 2, a_2\mid \mathrm {lcm}(a_2, a_{6})$ and $\mathrm {lcm}(a_2, a_{6})\ge a_6\ge 6$ , we deduce that $\mathrm {lcm}(a_2, a_{6})\ge 6$ with equality if and only if $a_2=2$ and $a_6=6$ . So

(2.3) $$ \begin{align} S_{6}=\frac{1}{\mathrm{lcm}(a_1, a_{5})} +\frac{1}{\mathrm{lcm}(a_2, a_{6})} \le\frac{1}{5}+\frac{1}{6}=\frac{11}{30}, \end{align} $$

with equality in (2.3) if and only if $\mathrm {lcm}(a_1, a_{5})=5$ and $\mathrm {lcm}(a_2, a_{6})=6$ , which is true if and only if $a_1=1, a_2=2, a_5=5$ and $a_6=6$ , which is true if and only if $a_i=i$ for all $i\in \{1,2,3,4, 5, 6\}$ .

Finally, we consider $S_7$ . Since $a_3\ge 3, a_3\mid \mathrm {lcm}(a_3, a_{7})$ and $\mathrm {lcm}(a_3, a_{7})\ge a_7\ge 7$ , we deduce that either $\mathrm {lcm}(a_3, a_{7})=8$ which is true if and only if $a_3=4$ and $a_7=8$ , or $\mathrm {lcm}(a_3, a_{7})=9$ which is true if and only if $a_3=3$ and $a_7=9$ , or $\mathrm {lcm}(a_3, a_{7})\ge 10$ . We divide the rest of the proof into three cases.

If $\mathrm {lcm}(a_3, a_{7})\ge 10$ , then

$$ \begin{align*} S_{7}=\frac{1}{\mathrm{lcm}(a_1, a_{5})} +\frac{1}{\mathrm{lcm}(a_2, a_{6})}+\frac{1}{\mathrm{lcm}(a_3, a_{7})} \le\frac{11}{30}+\frac{1}{10}<\frac{43}{90} \end{align*} $$

as desired.

If $\mathrm {lcm}(a_3, a_{7})=8$ , then $a_3=4$ and $a_7=8$ . This implies that $a_4=5, a_5=6$ and $a_6=7$ . Since $(a_1, a_2)\in \{(1,2), (1,3), (2,3)\}$ , we have $\mathrm {lcm}(a_1, a_{5})=6$ and $\mathrm {lcm}(a_2, a_{6})\in \{14, 21\}$ . It then follows that

$$ \begin{align*} S_{7}\le\frac{1}{6}+\frac{1}{14}+\frac{1}{8}<\frac{43}{90}. \end{align*} $$

If $\mathrm {lcm}(a_3, a_{7})=9$ , then we must have $a_3=3$ and $a_7=9$ . So $\mathrm {lcm}(a_3, a_{7})=9$ . It then follows that

(2.4) $$ \begin{align} S_{7}=S_6+\frac{1}{\mathrm{lcm}(a_3,a_7)} \le\frac{11}{30}+\frac{1}{9}=\frac{43}{90}, \end{align} $$

with equality in (2.4) if and only if $a_i=i$ for all $i\in \{1,2,3,4,5,6\}$ and $\mathrm {lcm}(a_3, a_{7})=9$ , if and only if $a_i=i$ for all $i\in \{1,2,3,4,5,6\}$ and $a_7=9$ as required.

This completes the proof of Lemma 2.3.

Lemma 2.4. Let m be a positive integer with $m\ge 2$ and $\mathcal {A}=\{a_i\}_{i=1}^8$ a strictly increasing sequence of eight positive integers. Let

(2.5) $$ \begin{align} \Box_m=\Box_m(\mathcal {A}):=\sum_{i=1}^4 \bigg(\frac{1}{\mathrm{lcm}(a_i, a_{i+4})} +\frac{1}{a_{i+4}}\bigg(1-\frac{1}{2^{m-2}}\bigg)\bigg). \end{align} $$

Then both of the following statements are true.

  1. (i) Either $\Box _2= {101}/{180}$ which is true if and only if $a_i=i$ for all $i\in \{1,2,3,4,5,6\}$ , $a_7=9$ and $a_8=12$ , or $\Box _2= {389}/{720}$ which holds if and only if $a_i=i$ for all integers $i\in \{1,2,3,4,5,6\}$ , $a_7=9$ and $a_8=16$ , or $\Box _2= {453}/{840}$ which is true if and only if $a_i=i$ for all integers $i\in \{1,2,3,4,5,6,7,8\}$ , or $\Box _2 < {453}/{840}$ .

  2. (ii) If $m\ge 3$ , then

    (2.6) $$ \begin{align} \Box_m\le\frac{493}{420}-\frac{533}{105}\cdot\frac{1}{2^{m+1}}, \end{align} $$
    with equality in (2.6) if and only if $a_i=i$ for all integers i with $1\le i\le 8$ .

Proof (i). Evidently, $\Box _2=\sum _{i=1}^4 {1}/{\mathrm {lcm}(a_i, a_{i+4})}.$ We consider the following cases.

Case 1: $a_5\ge 6$ . Then $a_8\ge 9$ . If $a_8\ge 10$ , then by the fact $\mathrm {lcm}(a_i, a_{i+4})\ge a_{i+4}$ for all $i\in \{1,2,3,4\}$ , we derive

$$ \begin{align*} \Box_2\le \frac{1}{a_5}+\frac{1}{a_6}+\frac{1}{a_7}+\frac{1}{a_8} \le \frac{1}{6}+\frac{1}{7}+\frac{1}{8}+\frac{1}{10} <\frac{1}{5}+\frac{1}{6}+\frac{1}{8}+\frac{1}{21}=\frac{453}{840}. \end{align*} $$

If $a_8=9$ , then $a_5=6$ , $a_6=7$ and $a_7=8$ . This implies that $a_1\in \{1,2\}$ , $a_2\in \{2, 3\}, a_3\in \{3, 4\}$ and $a_4\in \{4,5\}$ . It follows that $\mathrm {lcm}(a_1, a_5)=6$ , $\mathrm {lcm}(a_2, a_6)\in \{14, 21\}, \mathrm {lcm}(a_3, a_7)=\mathrm {lcm}(a_3, 8)\in \{8, 24\}$ and $\mathrm {lcm}(a_4, a_8)=\mathrm {lcm}(a_4, 9)\in \{36, 45\}$ . So

$$ \begin{align*} \Box_2\le\frac{1}{6}+\frac{1}{14}+\frac{1}{8}+\frac{1}{36} <\frac{1}{5}+\frac{1}{6}+\frac{1}{8}+\frac{1}{21}=\frac{453}{840}. \end{align*} $$

Case 2: $a_5=5$ . Then $a_i=i$ for all integers i with $1\le i\le 4$ . If $a_6\ge 7$ , then $a_7\ge 8$ and $a_8\ge 9$ . So $\mathrm {lcm}(a_1, a_5)=5, \mathrm {lcm}(a_2, a_6)\ge 8, \mathrm {lcm}(a_3, a_7)\ge 9$ and $\mathrm {lcm}(a_4, a_8)\ge 12$ . However, $1/9+ {1}/{12} < 1/6 + {1}/{21}$ . Thus

$$ \begin{align*} \Box_2\le\frac{1}{5}+\frac{1}{8}+\frac{1}{9}+\frac{1}{12} <\frac{1}{5}+\frac{1}{6}+\frac{1}{8}+\frac{1}{21}=\frac{453}{840}. \end{align*} $$

In what follows, we let $a_6=6$ . If $a_7\ge 10$ , then $a_8\ge 11$ . It follows that $\mathrm {lcm}(a_3, a_7)\ge 12$ with equality holding if and only if $a_7=12$ , and $\mathrm {lcm}(a_4, a_8)\ge 12$ with equality occurring if and only if $a_8=12$ . Since $a_7<a_8$ ,

$$ \begin{align*} \Box_2<\frac{1}{5}+\frac{1}{6}+\frac{1}{12}+\frac{1}{12} <\frac{1}{5}+\frac{1}{6}+\frac{1}{8}+\frac{1}{21}=\frac{453}{840}. \end{align*} $$

It remains to consider the case $a_7\in \{7, 8, 9\}$ . We consider three subcases.

Subcase 2.1: $a_7=7$ . Then $\mathrm {lcm}(a_3, a_7)=21$ and $\mathrm {lcm}(a_4, a_8)=\mathrm {lcm}(4, a_8)\ge 8$ with equality if and only if $a_8=8$ . So

$$ \begin{align*}\Box_2\le \frac{1}{5}+\frac{1}{6} +\frac{1}{21}+\frac{1}{8}=\frac{453}{840}\end{align*} $$

with equality if and only if $a_i=i$ for all integers i with $1\le i\le 8$ .

Subcase 2.2: $a_7=8$ . Then $a_8\ge 9$ . Hence

$$ \begin{align*} \Box_2\le \frac{1}{5}+\frac{1}{6}+\frac{1}{24}+\frac{1}{12} <\frac{1}{5}+\frac{1}{6}+\frac{1}{8}+\frac{1}{21}=\frac{453}{840}. \end{align*} $$

Subcase 2.3: $a_7=9$ . Then $a_8\ge 10$ . It follows that either $\mathrm {lcm}(a_4, a_8)=12$ which is true if and only if $a_8=12$ , or $\mathrm {lcm}(a_4, a_8)=16$ which is true if and only if $a_8=16$ , or $\mathrm {lcm}(a_4, a_8)\ge 20$ . We then deduce that either

$$ \begin{align*} \Box_2=\frac{1}{5}+\frac{1}{6}+\frac{1}{9}+\frac{1}{12}=\frac{101}{180} \end{align*} $$

which is true if and only if $a_i=i$ for all $i\in \{1,2,3,4,5,6\}$ , $a_7=9$ and $a_8=12$ , or

$$ \begin{align*} \Box_2=\frac{1}{5}+\frac{1}{6}+\frac{1}{9}+\frac{1}{16}=\frac{389}{720} \end{align*} $$

which holds if and only if $a_i=i$ for all $i\in \{1,2,3,4,5,6\}$ , $a_7=9$ and $a_8=16$ , or

$$ \begin{align*} \Box_2\le \frac{1}{5}+\frac{1}{6}+\frac{1}{9}+\frac{1}{20} <\frac{1}{5}+\frac{1}{6}+\frac{1}{8}+\frac{1}{21}=\frac{453}{840} \end{align*} $$

as expected. This completes the proof of part (i).

(ii). Let $m\ge 3$ . Since $\mathrm {lcm}(a_i, a_{i+4})\ge a_{i+4}$ for all integers i with $1\le i\le 4$ ,

(2.7) $$ \begin{align} \Box_m &\le\sum_{i=1}^4\bigg(\frac{1}{a_{i+4}}+\frac{1}{a_{i+4}} \bigg(1-\frac{1}{2^{m-2}}\bigg)\bigg) =\bigg(2-\frac{1}{2^{m-2}}\bigg)\sum_{i=5}^8\frac{1}{a_i} \end{align} $$

with equality in (2.7) if and only if $a_i\mid a_{i+4}$ for all integers $i\in \{1, 2, 3, 4\}$ . Let $S_0:= {493}/{420} - {533}/{105} \cdot {1}/({2^{m+1}})$ . We divide the rest of the proof into two cases.

Case 1: $a_5\ge 6$ . Then $a_6\ge 7, a_7\ge 8$ and $a_8\ge 9$ . So by (2.7) and Lemma 2.2,

$$ \begin{align*} \Box_m & \le \bigg(2-\frac{1}{2^{m-2}}\bigg)\sum_{i=5}^8\frac{1}{a_i} \le \bigg(\frac{1}{6}+\frac{1}{7}+\frac{1}{8} +\frac{1}{9}\bigg)\bigg(2-\frac{1}{2^{m-2}}\bigg)\nonumber\\[4pt] & <\frac{1}{5}+\frac{1}{6}+\frac{1}{8}+\frac{1}{21} +\bigg(\frac{1}{5}+\frac{1}{6}+\frac{1}{7} +\frac{1}{8}\bigg)\bigg(1-\frac{1}{2^{m-2}}\bigg) =S_0 \end{align*} $$

since $m\ge 3$ . This gives the desired result for Case 1.

Case 2: $a_5=5$ . Then $a_i=i$ for all $i\in \{1, 2, 3, 4\}$ . We consider three subcases.

Subcase 2.1: $a_6=6$ . Then $a_7\ge 7$ and $\mathrm {lcm}(a_3, a_7)=\mathrm {lcm}(3, a_7)\ge 9$ . So

(2.8) $$ \begin{align} \Box_m & = \frac{1}{\mathrm{lcm}(1, 5)}+\frac{1}{\mathrm{lcm}(2, 6)} +\frac{1}{\mathrm{lcm}(3, a_{7})}+\frac{1}{\mathrm{lcm}(4, a_8)}\nonumber\\[6pt] &\quad +\bigg(\frac{1}{5}+\frac{1}{6}+\frac{1}{a_7}+\frac{1}{a_8}\bigg)\bigg(1-\frac{1}{2^{m-2}}\bigg). \end{align} $$

If $a_7=7$ , then it follows from $a_8\ge 8$ that $\mathrm {lcm}(4, a_8)\ge 8$ with equality if and only if $a_8=8$ . Therefore,

$$ \begin{align*} \Box_m & = \frac{1}{5}+\frac{1}{6}+\frac{1}{21}+\frac{1}{\mathrm{lcm}(4, a_8)}+\bigg(\frac{1}{5}+\frac{1}{6}+\frac{1}{7}+\frac{1}{a_8}\bigg)\bigg(1-\frac{1}{2^{m-2}}\bigg)\\[4pt] & \le \frac{1}{5}+\frac{1}{6}+\frac{1}{21}+\frac{1}{8}+\bigg(\frac{1}{5} +\frac{1}{6}+\frac{1}{7}+\frac{1}{8}\bigg)\bigg(1-\frac{1}{2^{m-2}}\bigg)=S_0 \end{align*} $$

with equality if and only if $a_i=i$ for all integers i with $1\le i\le 8$ .

If $a_7=8$ , then $a_8\ge 9$ and so $\mathrm {lcm}(4, a_8)\ge 12$ . Thus by (2.8),

$$ \begin{align*} \Box_m<\frac{1}{5}+\frac{1}{6}+\frac{1}{24}+\frac{1}{12} +\bigg(\frac{1}{5}+\frac{1}{6}+\frac{1}{8}+\frac{1}{9}\bigg) \bigg(1-\frac{1}{2^{m-2}}\bigg)<S_0. \end{align*} $$

If $a_7=9$ , then $\mathrm {lcm}(3, a_7)=9$ , $a_8\ge 10$ and so $\mathrm {lcm}(4, a_8)\ge 12$ . Since $m\ge 3$ and by Lemma 2.2,

$$ \begin{align*} \Box_m & <\frac{1}{5}+\frac{1}{6}+\frac{1}{9}+\frac{1}{12} +\bigg(\frac{1}{5}+\frac{1}{6}+\frac{1}{9}+\frac{1}{10}\bigg)\bigg(1-\frac{1}{2^{m-2}}\bigg) \\[4pt] & <\frac{1}{5}+\frac{1}{6}+\frac{1}{8}+\frac{1}{21} +\bigg(\frac{1}{5}+\frac{1}{6}+\frac{1}{7}+\frac{1}{8}\bigg)\bigg(1-\frac{1}{2^{m-2}}\bigg) =S_0. \end{align*} $$

If $a_7\ge 10$ , then $a_8\ge 11$ . Hence $\mathrm {lcm}(3, a_7)\ge 12$ with equality holding if and only if $a_7=12$ , and $\mathrm {lcm}(4, a_8)\ge 12$ with equality occurring if and only if $a_8=12$ . Since $a_7<a_8$ and $m\ge 3$ ,

$$ \begin{align*} \Box_m & <\frac{1}{5}+\frac{1}{6}+\frac{1}{12}+\frac{1}{12} +\bigg(\frac{1}{5}+\frac{1}{6}+\frac{1}{10}+\frac{1}{11}\bigg)\bigg(1-\frac{1}{2^{m-2}}\bigg) \\[4pt] & <\frac{1}{5}+\frac{1}{6}+\frac{1}{21}+\frac{1}{8}+\bigg(\frac{1}{5} +\frac{1}{6}+\frac{1}{7}+\frac{1}{8}\bigg)\bigg(1-\frac{1}{2^{m-2}}\bigg) =S_0. \end{align*} $$

Subcase 2.2: $a_6=7$ . Then $a_7\ge 8$ and $a_8\ge 9$ . So $\mathrm {lcm}(3, a_7)\ge 9$ with equality if and only if $a_7=9$ , and $\mathrm {lcm}(4, a_8)\ge 12$ with equality if and only if $a_8=12$ . Since ${1}/{14} + 1/9 + {1}/{12} < 1/6 + 1/8 + {1}/{21}$ , it then follows immediately that

$$ \begin{align*} \Box_m & =\frac{1}{\mathrm{lcm}(1, 5)}+\frac{1} {\mathrm{lcm}(2, 7)}+\frac{1}{\mathrm{lcm}(3, a_{7})} +\frac{1}{\mathrm{lcm}(4, a_{8})}\nonumber\\[4pt] &\quad +\bigg(\frac{1}{5}+\frac{1}{7}+\frac{1}{a_7}+ \frac{1}{a_8}\bigg)\bigg(1-\frac{1}{2^{m-2}}\bigg)\nonumber\\[4pt] & <\frac{1}{5}+\frac{1}{14}+\frac{1}{9} +\frac{1}{12}+\bigg(\frac{1}{5}+\frac{1}{7}+\frac{1}{8}+ \frac{1}{9}\bigg)\bigg(1-\frac{1}{2^{m-2}}\bigg)\nonumber\\[4pt] & <\frac{1}{5}+\frac{1}{6}+\frac{1}{21}+\frac{1}{8} +\bigg(\frac{1}{5}+\frac{1}{6}+\frac{1}{7} +\frac{1}{8}\bigg)\bigg(1-\frac{1}{2^{m-2}}\bigg)=S_0. \end{align*} $$

Subcase 2.3: $a_6\ge 8$ . Then $a_7\ge 9$ and $a_8\ge 10$ . Thus $\mathrm {lcm}(a_2, a_6)=\mathrm {lcm}(2, a_6)\ge 8$ , $\mathrm {lcm}(a_3, a_7) =\mathrm {lcm}(3, a_7)\ge 9$ and $\mathrm {lcm}(a_4, a_8)=\mathrm {lcm}(4, a_8)\ge a_8\ge 10$ which implies that $\mathrm {lcm}(a_4, a_8)\ge 12$ since $4\mid \mathrm {lcm}(a_4, a_8)$ . It then follows from the inequality $ 1/9 + {1}/{12} < 1/6 + {1}/{21}$ that

$$ \begin{align*} \Box_m & =\frac{1}{\mathrm{lcm}(1, 5)}+\frac{1} {\mathrm{lcm}(2, a_6)}+\frac{1}{\mathrm{lcm}(3, a_{7})} +\frac{1}{\mathrm{lcm}(4, a_{8})} \\[4pt] &\quad +\bigg(\frac{1}{5}+\frac{1}{a_6}+\frac{1}{a_7}+ \frac{1}{a_8}\bigg)\bigg(1-\frac{1}{2^{m-2}}\bigg) \\& \le \frac{1}{5}+\frac{1}{8}+\frac{1}{9}+\frac{1}{12}+\bigg(\frac{1}{5}+\frac{1}{8} +\frac{1}{9}+\frac{1}{10}\bigg)\bigg(1-\frac{1}{2^{m-2}}\bigg)\nonumber\\[4pt] & <\frac{1}{5}+\frac{1}{6}+\frac{1}{21}+\frac{1}{8}+\bigg(\frac{1}{5}+\frac{1}{6} +\frac{1}{7}+\frac{1}{8}\bigg)\bigg(1-\frac{1}{2^{m-2}}\bigg)=S_0. \end{align*} $$

This completes the proof of part (ii).

3 Proof of Theorem 1.1

Let $m\ge 2$ be an integer and let $\Box _m$ be defined as in (2.5). Then $\Box _2=S_8$ , so the results for parts (i) to (iv) follow from Lemmas 2.3 and 2.4. It remains to prove (v).

We first deal with the upper bounds for $S_9, S_{10}$ and $S_{11}$ . For $r\in \{1,2,3\}$ ,

$$ \begin{align*} S_{8+r}=\Box_2+\sum_{i=1}^r\frac{1}{\mathrm{lcm}(a_{4+i}, a_{8+i})}. \end{align*} $$

By Lemma 2.4, either $\Box _2= {101}/{180}$ which is true if and only if $a_i=i$ for all $i\in \{1,2,3,4,5,6\}$ , $a_7=9$ and $a_8=12$ , or $\Box _2= {389}/{720}$ which holds if and only if $a_i=i$ for all integers $i\in \{1,2,3,4,5,6\}$ , $a_7=9$ and $a_8=16$ , or $\Box _2= {453}/{840}$ which is true if and only if $a_i=i$ for all integers $i\in \{1,2,3,4,5,6,7,8\}$ , or $\Box _2 < {453}/{840}$ .

If $\Box _2 < {453}/{840}$ , then it follows from $\mathrm {lcm}(a_5, a_9)\ge 10, \mathrm {lcm}(a_6, a_{10})\ge 12$ and $\mathrm {lcm}(a_7, a_{11})\ge 14$ that

$$ \begin{align*} S_9 & <\frac{453}{840}+\frac{1}{\mathrm{lcm}(a_5, a_9)} \le \frac{453}{840}+\frac{1}{10}=\frac{537}{840}, \\[3pt] S_{10} & <\frac{453}{840}+\sum_{i=1}^2\frac{1}{\mathrm{lcm}(a_{4+i}, a_{8+i})} \le \frac{453}{840}+\frac{1}{10}+\frac{1}{12}=\frac{607}{840}, \\[3pt] S_{11} & <\frac{453}{840}+\sum_{i=1}^3\frac{1}{\mathrm{lcm}(a_{4+i}, a_{8+i})} \le \frac{453}{840}+\frac{1}{10}+\frac{1}{12}+\frac{1}{14}=\frac{667}{840}. \end{align*} $$

If $\Box _2= {101}/{180}$ , then by Lemma 2.4, we must have $a_i=i$ for all integers i with $1\le i\le 6$ , $a_7=9$ and $a_8=12$ . So $a_9\ge 13$ , $a_{10}\ge 14$ and $a_{11}\ge 15$ . This implies that $\mathrm {lcm}(a_5, a_9)=\mathrm {lcm}(5, a_9)\ge 15$ with equality if and only if $a_9=15$ , $\mathrm {lcm}(a_6, a_{10})=\mathrm {lcm}(6, a_{10})\ge 18$ with equality if and only if $a_{10}=18$ , and $\mathrm {lcm}(a_7, a_{11})=\mathrm {lcm}(9, a_{11})\ge 18$ with equality if and only if $a_{11}=18$ . Hence

$$ \begin{align*} S_9 & =\frac{101}{180}+\frac{1}{\mathrm{lcm}(a_5, a_9)} \le\frac{101}{180}+\frac{1}{15}=\frac{113}{180}<\frac{537}{840}, \\[3pt] S_{10} & =\frac{101}{180}+\sum_{i=1}^2\frac{1}{\mathrm{lcm}(a_{4+i}, a_{8+i})} \le\frac{101}{180}+\frac{1}{15}+\frac{1}{18}=\frac{123}{180}<\frac{607}{840}, \\S_{11} & =\frac{101}{180}+\sum_{i=1}^3\frac{1}{\mathrm{lcm}(a_{4+i}, a_{8+i})} <\frac{101}{180}+\frac{1}{15}+\frac{1}{18} +\frac{1}{18}=\frac{133}{180}<\frac{667}{840} \end{align*} $$

as desired.

If $\Box _2= {389}/{720}$ , then by Lemma 2.4, we must have $a_i=i$ for all integers i with $1\le i\le 6$ , $a_7=9$ and $a_8=16$ . So $a_9\ge 17$ , $a_{10}\ge 18$ and $a_{11}\ge 19$ which implies that $\mathrm {lcm}(a_5, a_9)=\mathrm {lcm}(5, a_9)\ge 20$ with equality if and only if $a_9=20$ , $\mathrm {lcm}(a_6, a_{10})=\mathrm {lcm}(6, a_{10})\ge 18$ with equality if and only if $a_{10}=18$ and $\mathrm {lcm}(a_7, a_{11})=\mathrm {lcm}(9, a_{11})\ge 27$ with equality if and only if $a_{11}=27$ . One then deduces that

$$ \begin{align*} S_9 & =\frac{389}{720}+\frac{1}{\mathrm{lcm}(a_5, a_9)} \le\frac{389}{720}+\frac{1}{20}=\frac{425}{720}<\frac{537}{840}, \\[4pt] S_{10}& =\frac{389}{720}+\sum_{i=1}^2\frac{1}{\mathrm{lcm}(a_{4+i}, a_{8+i})} <\frac{389}{720}+\frac{1}{20}+\frac{1}{18}=\frac{465}{720}<\frac{607}{840}, \\[4pt] S_{11} & =\frac{389}{720}+\sum_{i=1}^3\frac{1}{\mathrm{lcm}(a_{4+i}, a_{8+i})} \le\frac{389}{720}+\frac{1}{20}+\frac{1}{18} +\frac{1}{27}=\frac{465}{720}+\frac{1}{27}<\frac{667}{840} \end{align*} $$

as desired.

If $\Box _2 = {453}/{840}$ , then by Lemma 2.4, we must have $a_i=i$ for all integers i with $1\le i\le 8$ . So $a_9\ge 9$ which implies that $\mathrm {lcm}(a_5, a_9)\ge 10$ with equality if and only if $a_9=10$ . Furthermore, $\mathrm {lcm}(a_6, a_{10})\ge 12$ with equality if and only if $a_{10}=12$ and $\mathrm {lcm}(a_7, a_{11})\ge 14$ with equality if and only if $a_{11}=14$ . Thus

(3.1) $$ \begin{align} \hspace{-74pt}S_9 =\frac{453}{840}+\frac{1}{\mathrm{lcm}(a_5, a_9)} \le\frac{453}{840}+\frac{1}{10}=\frac{537}{840}, \end{align} $$
(3.2) $$ \begin{align} \hspace{-34pt}\ \ \ S_{10} =\frac{453}{840}+\sum_{i=1}^2\frac{1}{\mathrm{lcm}(a_{4+i}, a_{8+i})} \le\frac{453}{840}+\frac{1}{10}+\frac{1}{12}=\frac{607}{840}, \end{align} $$
(3.3) $$ \begin{align} S_{11} & =\frac{453}{840}+\sum_{i=1}^3\frac{1}{\mathrm{lcm}(a_{4+i}, a_{8+i})} \le\frac{453}{840}+\frac{1}{10}+\frac{1}{12}+\frac{1}{14}=\frac{667}{840}, \end{align} $$

where each equality in (3.1) to (3.3) holds if and only if $a_i=i$ for all integers i with $1\le i\le 8$ , $a_9=10$ , $a_{10}=12$ and $a_{11}=14$ . So part (v) is true when $9\le n\le 11$ .

In what follows, we always assume that $n\ge 12$ . Then we can write $n=4m$ or $n=4m+r$ for some integers m and r with $m\ge 3$ and $1\le r\le 3$ . For any integer i with $1\le i\le 4$ , we define

$$ \begin{align*} S_m^{(i)}:=\sum_{j=1}^{m-2}\frac{1}{\mathrm{lcm}(a_{4j+i}, a_{4j+4+i})}. \end{align*} $$

Then

(3.4) $$ \begin{align} S_{4m}&=\sum_{i=1}^4\bigg(\frac{1}{\mathrm{lcm}(a_i, a_{i+4})}+S_m^{(i)}\bigg) \end{align} $$

and

(3.5) $$ \begin{align} S_{4m+r}=S_{4m}+\sum_{i=1}^r\frac{1}{\mathrm{lcm}(a_{4m-4+i}, a_{4m+i})}. \end{align} $$

For any integer i with $1\le i\le 4$ , applying Lemma 2.1 to the subsequence $\{a_{i+4}, a_{i+8},\ldots , a_{i+4(m-1)}\}$ yields

(3.6) $$ \begin{align} S_m^{(i)}=\sum_{j=1}^{m-2}\frac{1}{\mathrm{lcm}(a_{i+4j}, a_{i+4j+4})} \le \frac{1}{a_{i+4}}\bigg(1-\frac{1}{2^{m-2}}\bigg) \end{align} $$

with equality in (3.6) if and only if $a_{i+4j}=a_{i+4}\times 2^{j-1}$ for all integers j with $1\le j\le m-1$ . Further, for any integer i with $1\le i\le r$ , applying Lemma 2.1 to the subsequence $\{a_{4+i}, a_{8+i},\ldots , a_{4m+i}\}$ gives

(3.7) $$ \begin{align} S_m^{(i)}+\frac{1}{\mathrm{lcm}(a_{4m-4+i}, a_{4m+i})} =\sum_{j=1}^{m-1}\frac{1}{\mathrm{lcm}(a_{4j+i}, a_{4j+4+i})} \le \frac{1}{a_{4+i}}\bigg(1-\frac{1}{2^{m-1}}\bigg) \end{align} $$

with equality in (3.7) if and only if $a_{4j+i}=a_{4+i}\times 2^{j-1}$ for all integers j with $1\le j\le m$ . Then by (3.4) and (3.6),

(3.8) $$ \begin{align} S_{4m} & \le \sum_{i=1}^4\bigg(\frac{1}{\mathrm{lcm}(a_i, a_{i+4})} +\frac{1}{a_{i+4}}\bigg(1-\frac{1}{2^{m-2}}\bigg)\bigg)=\Box_m \end{align} $$

with equality in (3.8) if and only if $a_{4j+i}=a_{4+i}\times 2^{j-1}$ for all integers i and j with $1\le j\le m-1$ and $1\le i\le 4$ . By (3.5), (3.6) and (3.7),

(3.9) $$ \begin{align} S_{4m+r}&=\sum_{i=1}^4\frac{1}{\mathrm{lcm}(a_i, a_{i+4})} +\sum_{i=1}^r\bigg(S_m^{(i)}+\frac{1}{\mathrm{lcm}(a_{4m-4+i}, a_{4m+i})}\bigg)+\sum_{i=r+1}^4 S_m^{(i)}\nonumber\\ &\le \sum_{i=1}^4\frac{1}{\mathrm{lcm}(a_i, a_{i+4})} +\sum_{i=1}^r \frac{1}{a_{4+i}}\bigg(1-\frac{1}{2^{m-1}}\bigg) +\sum_{i=r+1}^4 \frac{1}{a_{4+i}}\bigg(1-\frac{1}{2^{m-2}}\bigg)\nonumber \\ &=\sum_{i=1}^4\bigg(\frac{1}{\mathrm{lcm}(a_i, a_{i+4})}+\frac{1}{a_{i+4}}\bigg(1-\frac{1}{2^{m-2}}\bigg)\bigg) +\frac{1}{2^{m-1}}\sum_{i=1}^r \frac{1}{a_{4+i}}\nonumber\\ &=\Box_m+\frac{1}{2^{m-1}}\sum_{i=1}^r\frac{1}{a_{4+i}}, \end{align} $$

and equality in (3.9) holds if and only if $a_{4j+i}=a_{4+i}\times 2^{j-1}$ for all integers i and j with $1\le j\le m-1$ and $1\le i\le 4$ and $a_{4m+i}=a_{4+i}\times 2^{m-1}$ for all integers i with $1\le i\le r$ . Now by Lemma 2.4, if $m\ge 3$ , then

(3.10) $$ \begin{align} \Box_m\le &\frac{1}{5}+\frac{1}{6}+\frac{1}{8}+\frac{1}{21} +\bigg(\frac{1}{5}+\frac{1}{6}+\frac{1}{7}+\frac{1}{8}\bigg) \bigg(1-\frac{1}{2^{m-2}}\bigg)\notag\\[4pt] =&\frac{493}{420}-\frac{533}{105}\cdot\frac{1}{2^{m+1}}:=S_0, \end{align} $$

with equality in (3.10) if and only if $a_i=i$ for all integers i with $1\le i\le 8$ . Notice that

(3.11) $$ \begin{align} \sum_{i=1}^r\frac{1}{a_{4+i}}\le\sum_{i=1}^r\frac{1}{4+i} \end{align} $$

with equality in (3.11) if and only if $a_{4+i}=4+i$ for all $1\le i\le r$ . Therefore, by (3.8) and (3.10), $S_{4m}\le S_0$ with equality if and only if $a_i=i$ for all $i\in \{1, 2, 3, 4\}$ and $a_{4j+i}=(4+i)\times 2^{j-1}$ for all integers i and j with $1\le j\le m-1$ and $1\le i\le 4$ . It follows from (3.9) and (3.11) that

(3.12) $$ \begin{align} S_{4m+r}\le S_0+\frac{1}{2^{m-1}}\sum_{i=1}^r\frac{1}{4+i} =\frac{493}{420}-\frac{533}{105}\cdot\frac{1}{2^{m+1}} +\frac{1}{2^{m-1}}\sum_{i=1}^r\frac{1}{4+i}, \end{align} $$

with equality in (3.12) if and only if $a_i=i$ for all $i\in \{1, 2, 3, 4\}$ , $a_{4j+i}=(4+i)\times 2^{j-1}$ for all integers i and j with $1\le j\le m-1$ and $1\le i\le 4$ and $a_{4m+i}=(4+i)\times 2^{m-1}$ for $1\le i\le r$ . So part (v) is proved when $n\ge 12$ .

This completes the proof of Theorem 1.1.

Acknowledgement

This work was carried out during a visit by the first author to Brock University as a postdoctoral fellow. He would like to sincerely thank the host institution for its hospitality and for providing an excellent atmosphere for research.

Footnotes

This work was supported in part by research grants from the Natural Sciences and Engineering Research Council of Canada (Grant No. DDG-2019-04206 and RGPIN 2017-03903).

References

Bateman, P., Kalb, J. and Stenger, A., ‘A limit involving least common multiples’, Amer. Math. Monthly 109 (2002), 393394.CrossRefGoogle Scholar
Behrend, F. A., ‘Generalization of an inequality of Heilbronn and Rohrbach,’ Bull. Amer. Math. Soc. (N.S.) 54 (1948), 681684.CrossRefGoogle Scholar
Borwein, D., ‘A sum of reciprocals of least common multiples’, Canad. Math. Bull. 20 (1978), 117118.CrossRefGoogle Scholar
Chebyshev, P. L., ‘Memoire sur les nombres premiers’, J. Math. Pures Appl. (9) 17 (1852), 366390.Google Scholar
Erdős, P. and Selfridge, J., ‘The product of consecutive integers is never a power’, Illinois J. Math. 19 (1975), 292301.CrossRefGoogle Scholar
Farhi, B., ‘Nontrivial lower bounds for the least common multiple of some finite sequences of integers’, J. Number Theory 125 (2007), 393411.CrossRefGoogle Scholar
Goutziers, C. J., ‘On the least common multiple of a set of integers not exceeding  $N$ ’, Indag. Math. (N.S.) 42 (1980), 163169.CrossRefGoogle Scholar
Hanson, D., ‘On the product of the primes’, Canad. Math. Bull. 15 (1972), 3337.CrossRefGoogle Scholar
Heilbronn, H. A., ‘On an inequality in the elementary theory of numbers’, Math. Proc. Cambridge Philos. Soc. 33 (1937), 207209.CrossRefGoogle Scholar
Hong, S. A., ‘A sharp upper bound for the sum of reciprocals of least common multiples’, Acta Math. Hungar. 160 (2020), 360375.CrossRefGoogle Scholar
Hong, S. F., Luo, Y. Y., Qian, G. Y. and Wang, C. L., ‘Uniform lower bound for the least common multiple of a polynomial sequence’, C. R. Math. Acad. Sci. Paris Ser. I 351 (2013), 781785.CrossRefGoogle Scholar
Hong, S. F., Qian, G. Y. and Tan, Q. R., ‘The least common multiple of a sequence of products of linear polynomials’, Acta Math. Hungar. 135 (2012), 160167.CrossRefGoogle Scholar
Ireland, K. and Rosen, M., A Classical Introduction to Modern Number Theory, Graduate Texts in Mathematics, 84 (Springer-Verlag, New York, 1990).CrossRefGoogle Scholar
Nair, M., ‘On Chebyshev-type inequalities for primes’, Amer. Math. Monthly 89 (1982), 126129.CrossRefGoogle Scholar
Rohrbach, H., ‘Beweis einer zahlentheoretischen Ungleichung’, J. reine angew. Math. 177 (1937), 193196.CrossRefGoogle Scholar